1.6 Answer

1.6

1.

(a) $\displaystyle{\lim_{n \to \infty} n^4 - 3n^3 = \lim_{n \to \infty}n^{4}(1 - \frac{3}{n}) = \infty}$

(b) $\displaystyle{\lim_{n \to \infty}\frac{3n^2 + 5}{4n^3 - 1} = \lim_{n \to \infty}\frac{n^2(3 + \frac{5}{n^2})}{n^3(4 - \frac{1}{n^3})} = 0}$

(c) $\displaystyle{\lim_{n \to \infty}\frac{1 - n}{n - \sqrt{n}} = \lim_{n \to \infty}\frac{n(\frac{1}{n} - 1)}{n(1 - \frac{\sqrt{n}}{n})} = -1}$

(d)

$\displaystyle \lim_{n \to \infty}\frac{n(n+2)}{n+1} - \frac{n^3}{n^2 + 1}$ $\displaystyle =$ $\displaystyle \lim_{n \to \infty}\frac{(n^2 + 2n)(n^2 + 1) - n^3(n+1)}{(n+1)(n^2 + 1)}$  
  $\displaystyle =$ $\displaystyle \lim_{n \to \infty}\frac{n^3 + n^2 + 2n}{n^3 + n^2 + n+ 1}$  
  $\displaystyle =$ $\displaystyle \lim_{n \to \infty}\frac{n^3(1 + \frac{1}{n} + \frac{2}{n^2})}{n^3(1 + \frac{1}{n} + \frac{1}{n^2} + \frac{1}{n^3})} = 1$  

(e)

$\displaystyle \lim_{n \to \infty}\sqrt{n+1} - \sqrt{n}$ $\displaystyle =$ $\displaystyle \lim_{n \to \infty}\frac{n+1 - n}{(\sqrt{n+1} + \sqrt{n})} = 0$  

2.

Note that for $a > 1$, we have $\sqrt[n]{a} > 1$. Thus, we can set $\sqrt[n]{a} = 1 + h, \ h > 0$. Then

$\displaystyle a = (1 + h)^{n} = 1 + nh + \cdots + h^n \geq 1 + nh $

Thus,

$\displaystyle \lim_{n \rightarrow \infty}h = \lim_{n \rightarrow \infty}\frac{a - 1}{n} = 0 $

For $a = 1$Cwe have $\sqrt[n]{a} = 1$ for all $n$. Thus,

$\displaystyle \lim_{n \rightarrow \infty}\sqrt[n]{a} = 1 $

For $0 < a < 1$, we let $\displaystyle{b = \frac{1}{a}}$. Then $b > 1$ and

$\displaystyle 1 = \lim_{n \rightarrow \infty}\sqrt[n]{b} = \lim_{n \rightarrow \infty}\frac{1}{\sqrt[n]{a}} $

3.

(a) $0 < b < a$ implies $a^{n} < a^{n} + b^{n} < 2a^{n}$D Then, $\displaystyle{a < (a^{n} + b^{n})^{\frac{1}{n}} < 2^{\frac{1}{n}} a}$D Thus, $\displaystyle{\lim_{n \to \infty}(a^{n} + b^{n})^{\frac{1}{n}} = a}$.

(b) Since $\displaystyle{3^{n} \leq 1 + 2^{n} + 3^{n} \leq 3 \dot 3^{n}}$, we take $n$th root. Then $\displaystyle{3 \leq (1 + 2^{n} + 3^{n})^{\frac{1}{n}} \leq 3^{\frac{1}{n}} \dot 3}$DAs $n \to \infty$, $3^{\frac{1}{n}} \to 1$. Then by squeezing theorem, we have $\displaystyle{\lim_{n \to \infty}(1 + 2^{n} + 3^{n})^{\frac{1}{n}} = 3}$